¿El argumento de Dirac contra la mecánica clásica está en contradicción con la teoría de Bohm?

En su libro sobre Mecánica Cuántica, PAM Dirac habla de la estabilidad del átomo como un medio para demostrar la necesidad de la mecánica cuántica. El escribe:

Los resultados experimentales muestran claramente la necesidad de apartarse de la mecánica clásica. En primer lugar, las fuerzas conocidas en la electrodinámica clásica son inadecuadas para explicar la notable estabilidad de los átomos y las moléculas, que es necesaria para que los materiales puedan tener propiedades físicas y químicas definidas. La introducción de nuevas fuerzas hipotéticas no salvará la situación, ya que existen principios generales de la mecánica clásica, válidos para todo tipo de fuerzas, que conducen a resultados en desacuerdo directo con la observación. Por ejemplo, si un sistema atómico tiene su equilibrio perturbado de alguna manera y luego se deja solo, se pondrá en oscilación y las oscilaciones quedarán impresas en el campo electromagnético circundante, de modo que sus frecuencias se puedan observar con un espectroscopio. Ahora, sean cuales sean las leyes de fuerza que gobiernan el equilibrio, uno esperaría poder incluir las diversas frecuencias en un esquema que comprende ciertas frecuencias fundamentales y sus armónicos. No se observa que este sea el caso. En cambio, se observa una nueva e inesperada conexión entre las frecuencias, llamada Ley de Combinación de la Espectroscopia de Ritz, según la cual todas las frecuencias pueden expresarse como diferencias entre ciertos términos, siendo el número de términos mucho menor que el número de frecuencias. Esta ley es bastante ininteligible desde el punto de vista clásico. se observa una nueva e inesperada conexión entre las frecuencias, llamada Ley de Combinación de la Espectroscopia de Ritz, según la cual todas las frecuencias pueden expresarse como diferencias entre ciertos términos, siendo el número de términos mucho menor que el número de frecuencias. Esta ley es bastante ininteligible desde el punto de vista clásico. se observa una nueva e inesperada conexión entre las frecuencias, llamada Ley de Combinación de la Espectroscopia de Ritz, según la cual todas las frecuencias pueden expresarse como diferencias entre ciertos términos, siendo el número de términos mucho menor que el número de frecuencias. Esta ley es bastante ininteligible desde el punto de vista clásico.

Dado que la teoría de De-Broglie Bohm esencialmente introduce una fuerza cuántica para remediar la mecánica clásica, ¿el argumento de Dirac está en contradicción con la teoría de Bohm?

Encontrará que hay bastante menos que aceptación universal de la teoría de de Broglie Bohm. Tiene sus adeptos (algunos de los cuales insisten en que obviamente es mejor que otras interpretaciones), pero también sus detractores (quienes a menudo son iguales y están seguros de que es la interpretación menos probable y claramente se inventó bajo la influencia de drogas fuertes). ). También hay muchas, muchas personas a las que simplemente no les importan las batallas interpretativas a menos que y hasta que se proponga un experimento discriminatorio.

Respuestas (3)

Dirac no pretende que clásica signifique mecánica no cuántica ; él pretende clásica para significar pre- mecánica cuántica 1 . Así que no, esto no dice nada sobre la teoría de De Broglie-Bohm.

Dirac abre su párrafo con

Los resultados experimentales muestran claramente la necesidad de apartarse de la mecánica clásica.

Dirac no está hablando de ninguna teoría que hoy se pueda llamar "clásica". Está hablando de la teoría de la mecánica clásica que existía antes de que se formulara la mecánica cuántica. Si lees su párrafo, todas sus críticas están dirigidas contra esa teoría en particular. Por ejemplo, dice

las fuerzas conocidas en la electrodinámica clásica

y

si un sistema atómico tiene su equilibrio perturbado de alguna manera y luego se lo deja solo, se pondrá en oscilación y las oscilaciones quedarán impresas en el campo electromagnético circundante.

Estas propiedades son específicas de la mecánica clásica precuántica. Por lo tanto, su argumento no dice nada sobre la teoría de De Broglie-Bohm. De hecho, sospecho que si le hubieras preguntado a Dirac, habría dicho que la teoría de De Broglie-Bohm no era mecánica clásica.

EDITAR: Quiero abordar los comentarios de Lubos, porque algunas personas parecen estar de acuerdo con él y votarme negativamente. Además, en los comentarios me pidieron que explicara por qué la crítica de Dirac no se aplica a la mecánica de Bohm. Si crees que Lubos es un genio y tomas todo lo que dice sin críticas, sigue adelante y vota a la baja. Si realmente puede razonar críticamente y pensar de manera independiente, lea mi refutación a los comentarios de Lubos antes de votar negativamente.

Lubos se equivoca en casi todo lo que dice sobre la teoría de Bohm en sus comentarios. Lubos afirma que la onda piloto de la teoría de Bohm es observable. La onda piloto en la teoría de Bohm es exactamente igual que la función de onda de Schroedinger, por lo que si la función de onda en la mecánica cuántica no es observable, tampoco lo es la onda piloto en la mecánica de Bohm. Además, la teoría de Bohm tiene exactamente las mismas predicciones que la mecánica cuántica, por lo que está claro que en la teoría de Bohm, un sistema no "se pondrá en oscilación y las oscilaciones quedarán impresas en el campo electromagnético circundante", simplemente porque no es en la mecánica cuántica estándar.

La única forma que veo para salvar las afirmaciones de Lubos es darme cuenta de que la teoría de Bohm en realidad no funciona relativistamente 2 y luego razonar que debido a que los fotones son intrínsecamente relativistas, los fotones en realidad no funcionan en ella. Pero a pesar de que Dirac era un hombre increíblemente inteligente, es completamente ridículo afirmar que

  1. Dirac pretendía que su argumento funcionara para todas las teorías no cuánticas y
  2. previó la teoría de Bohm y fue lo suficientemente inteligente como para darse cuenta de que su argumento se salvaría por el hecho de que la teoría de Bohm no funciona de manera relativista.

Dirac fue lo suficientemente inteligente como para saber que no se puede probar un teorema como este (es decir, ninguna teoría excepto la mecánica cuántica puede explicar la observación) sin establecer las hipótesis. Sus hipótesis, que estoy seguro pensó que había establecido claramente, eran que la física se comporta de acuerdo con las líneas de la física anterior a 1900... por ejemplo, la definición del diccionario de clásico como tradicional en estilo y forma . En 1930, no creo que nadie estuviera usando "física clásica" en el sentido de "no cuántico". (Me retractaré de esta declaración si puede encontrar un ejemplo de "clásico" que signifique claramente "no cuántico" en lugar de "precuántico" antes de 1940). Estaban usando "clásico" para referirse a "física anterior a 1900", es decir,

1 Una definición estándar de clásico es: tradicional en estilo o forma.

2 Los partidarios de la teoría de Bohm pueden afirmar lo contrario, pero no creo que nadie haya demostrado que se pueda tener una teoría de Bohm con trayectorias de partículas invariantes de Lorentz y relatividad general, lo que significa que, relativistamente, la teoría tiene graves inconvenientes.

No, "clásico" y "precuántico" son exactamente sinónimos, y el significado de estos adjetivos no ha cambiado ni un ápice desde la publicación del libro de Dirac. El argumento de Dirac es un argumento meritocrático que solo depende de las características intrínsecas de la teoría y no del año en que alguien propuso una teoría u otras irrelevancias sociológicas. Cuando uno analiza el argumento, es espectacularmente obvio que también se aplica a la teoría de Bohm porque es una teoría clásica de acuerdo con la terminología relevante y casi rigurosa que él fue muy cuidadoso en seguir.
Por lo tanto, está afirmando que esta afirmación: "Si un sistema atómico tiene su equilibrio perturbado de alguna manera y luego se lo deja solo, se pondrá en oscilación y las oscilaciones quedarán impresas en el campo electromagnético circundante" se aplica a cualquier teoría de física que obedece a las ecuaciones de Bell? ¿¿¿¿¿Por qué?????
Por supuesto, se aplica a cualquier teoría clásica, incluida la mecánica de Bohm (la de Bell fue un error tuyo, ¿no?). Se sigue de la definición de un equilibrio. Cualquier objeto en una teoría clásica que incluya la mecánica de Bohm tiene algunos estados puros que se representan en el equilibrio. Y también tiene deformaciones arbitrariamente débiles de este estado. La respuesta del campo electromagnético es una función continua de las coordenadas de fase del objeto, por lo que una pequeña deformación conducirá inevitablemente a huellas débiles en el campo electromagnético.
Si el estado es estable, se sigue que la primera derivada de la energía con respecto a la coordenada es cero y la segunda derivada es positiva: es un mínimo, no un máximo. Un máximo implicaría una inestabilidad. Con la segunda derivada positiva, los grados de libertad oscilan como un oscilador armónico. No hay forma de evitarlo. En cualquier teoría clásica, las perturbaciones débiles se traducen en huellas débiles en la radiación, tal como él lo dice.
La única razón por la que los bohmianos no se dan cuenta activamente de que este es obviamente el caso de una teoría bohmiana también es que nunca se han atrevido a pensar en cómo la teoría del tipo Bohmiano podría describir un objeto en equilibrio. Saben que nada de lo que han discutido es lo suficientemente poderoso como para hacer tales cosas. Pero si se escribe una teoría bien definida sin el principio de incertidumbre, por supuesto que no puede evitar los hechos generales de la física clásica y el tratamiento de Dirac, y se siguen las mismas conclusiones. Pueden ser inconvenientes, pero son inevitables.
La mecánica bohmiana es exactamente el tipo de teorías que Dirac excluye inmediatamente en la primera página de su libro Los principios de la mecánica cuántica. ¿Has leído al menos la primera página del libro? No parece tan Discute explícitamente por qué los nuevos tipos de fuerzas no ayudarán de ninguna manera: esto estaba destinado exactamente a lidiar con fuerzas como la fuerza de la onda piloto. Solo léalo, verá el argumento de por qué estas teorías no pueden funcionar.
La onda piloto afecta a la primera derivada de la posición de la partícula bohmiana, no a la segunda, y en este sentido, todo se expande al igual que la onda piloto se diluye. Entonces puedes pensar que evitas las oscilaciones sinusoidales. Pero es solo porque la evolución está lejos de cualquier equilibrio. Si uno se acerca a un equilibrio, el objeto matemático que describe el equilibrio simplemente tiene que ser descrito por una ecuación equivalente al oscilador armónico, efectivamente una ecuación de segundo orden. En física no cuántica, la ecuación diferencial de primer orden no permite ningún equilibrio.
Ecuación de primer orden - Schr. ecuación: en QM solo puede describir objetos cerca del equilibrio porque la función que ingresa a la ecuación no es observable, es el vector puro compuesto de amplitudes de probabilidad. Pero todas las ecuaciones para los observables reales, las ecuaciones de Heisenberg, pueden combinarse y eliminarse para obtener ecuaciones de segundo orden para observables similares a x alrededor de cualquier punto de equilibrio, al igual que las ecuaciones para los límites clásicos de estos observables.
@Lubos: De Dirac: en la primera oración, Dirac define efectivamente la mecánica clásica: "La mecánica clásica se ha desarrollado continuamente desde la época de Newton y se ha aplicado a una gama cada vez mayor de sistemas dinámicos, incluido el campo electromagnético en interacción con la materia ." No está hablando de mecánica no cuántica . Está hablando de la mecánica pre-cuántica . Y escribió esto en 1930, cuando Bohm tenía 13 años. No pretendía ser una refutación de la mecánica de Bohm.
No, la definición dice claramente que la mecánica clásica significa mecánica no cuántica, y la palabra precuántica, si alguien la usa, también significa exactamente lo mismo. Ningún físico del calibre de Dirac ha usado jamás la palabra "física clásica" de manera diferente. "Clásico" significa "no cuántico". Todo el mundo en física de partículas o materia condensada le dirá eso.
La segunda oración del libro de Dirac define la física clásica: "La tradición clásica ha sido considerar el mundo como una asociación de objetos observables (partículas, fluidos, campos, etc.) que se mueven de acuerdo con leyes de fuerza definidas, de modo que uno podría formar un imagen mental en el espacio y el tiempo de todo el esquema" Está espectacularmente claro que esto incluye la mecánica de Bohm, ¿no es así? Incluso si no lo llamaste "física clásica", no importa. Todavía obedece a las propiedades matemáticas para que puedan aplicarse las herramientas y los teoremas de la física clásica, tal como los describe Dirac.
@Lubos: no es "espectacularmente claro" para mí cómo una asociación de objetos observables podría incluir una onda piloto no observable.
Estimado Peter, se afirma que la onda piloto "guía" a la partícula y, en cualquier caso, los aspectos de la onda piloto o de la partícula bohmiana se observan experimentalmente en última instancia. Entonces, ¿cómo puedes decir que la onda piloto es "no observable"? Y ninguna parte de la onda piloto (que matemáticamente es la función de onda) es "físicamente redundante"; de lo contrario, ya se habría omitido. Todos ellos afectan las observaciones, fui tan generoso que escribí una introducción especial a la Página 1 de Dirac, busque en Google "Lentamente para Peter Shor: Página 1 de Dirac", pero ¿puedo dejar el resto para otros educadores?
@PeterShor Tengo que estar de acuerdo con Lubos en esto. Simplemente mencionas que la teoría de Bohm vino después del libro de Dirac, pero eso no significa nada. Si escribo una teoría en 2016 afirmando que la ley del cuadrado inverso para la gravitación debería ser en realidad una ley cúbica inversa, no significa que la ley centenaria sea incorrecta simplemente porque no podría haber sabido sobre mi trabajo en 2016. En cambio, significa que soy un mal teórico. Si cree que la teoría de Bohm escapa al comentario de Dirac, muestre algunos cálculos reales de cómo logra esto.
@ user7348: Creo que he demostrado por qué el comentario de Dirac no se aplica a la teoría de Bohm.
@PeterShor Realmente me refería a la dificultad de calcular las capacidades de calor en la teoría de Bohm. Motl ha señalado que incluso la aditividad de las capacidades caloríficas no se mantiene en BM. Si crees que realmente puedes realizar el k B cálculo requerido de cualquier teoría física correcta, estaría muy interesado en verlo. De hecho, responda mi pregunta: physics.stackexchange.com/questions/256343/…
@user7348: Motl no entiende la teoría de Bohm en absoluto, y está argumentando en contra de un hombre de paja: lo que él cree que es la teoría de Bohm en lugar de lo que realmente es. Sospecho que la termodinámica es muy difícil de hacer en la teoría de Bohm, ya que parece ser difícil definir microestados. Realmente no me gusta la teoría de Bohm y ciertamente no voy a tratar de calcular las capacidades de calor en ella.
@PeterShor Soy consciente de que Lubos no tiene ni idea, todo el mundo lo sabe, tal vez incluso el propio Motl. Gracias por la discusión.
@PeterShor Además, lamento que su respuesta a mi pregunta haya llevado a Lubos a escribir una publicación de blog completa insultando su inteligencia. Es completamente poco profesional e inapropiado.
@user7348: En realidad, la publicación del blog de Lubos ocurrió antes de mi respuesta a su pregunta.
Siento que algunas de estas discusiones sobre Stackexchange están ambientadas en la década de 1980 y no tienen en cuenta la evidencia experimental reciente: los experimentadores utilizaron mediciones débiles para hacer observaciones directas de las funciones de onda cuánticas en 2011.

Sí, absolutamente, el argumento de Dirac muestra que uno nunca podría construir una teoría completa, que especifique las reglas para la evolución, así como las predicciones para la medición y lo que sucede después de la medición, que sea compatible con los hechos básicos sobre los átomos.

Este teorema de no-go se aplica a la mecánica de Bohm porque, de hecho, es solo otra teoría clásica. La mecánica de Bohm es una teoría clásica compuesta por un campo clásico ψ , la onda piloto, que se dice que obedece a las mismas ecuaciones que la función de onda en la mecánica cuántica, pero tiene una interpretación completamente diferente, además de algunas funciones adicionales, típicamente posiciones de partículas clásicas. X i ( t ) . Estas posiciones están influenciadas por la onda piloto pero no existe la reacción opuesta.

Como cualquier sistema clásico lo suficientemente normal, el espacio de configuración de la mecánica de Bohm es continuo, por lo que siempre se puede dividir una pequeña perturbación por dos y lo que esta perturbación evoluciona a partir de esta perturbación es el resultado original dividido por dos también.

Eso está en contradicción directa con hechos tales como los espectros discretos de los átomos, etc. (Los estados propios de energía de los átomos pueden estar presentes matemáticamente, pero la mecánica de Bohm no permitirá ningún mecanismo que pueda imprimir los espectros a la radiación electromagnética de la luz emitida). La ecuación dinámica (es decir, la ecuación de Schrödinger en particular) en la mecánica cuántica es continua, pero el significado de la función de onda es probabilístico, por lo que una perturbación muy pequeña de ψ significa una probabilidad muy pequeña de un nuevo proceso nuevo (pero finitamente fuerte), no una presencia garantizada de un proceso muy débil. Esta es una clave diferente de la mecánica cuántica de cualquier teoría clásica, incluida la mecánica de Bohm.

El libro de texto de Dirac comienza con varios otros argumentos que descartan instantáneamente las teorías clásicas, incluida la mecánica de Bohm, especialmente el argumento sobre la baja capacidad calorífica del observador de todos los átomos . Si la teoría de Bohm pudiera extenderse a una teoría completa de los átomos que interactúan, se describirían mediante un gran espacio de fase porque tienen muchos grados de libertad clásicos y nuevos. Si el equilibrio térmico fuera posible en tal teoría, la entropía de un átomo (el logaritmo del volumen del espacio de fase de estados que son accesibles en el equilibrio térmico) sería enorme debido a estos grados de libertad adicionales, en forma directa. conflicto con los experimentos (que muestra que la capacidad de calor del átomo es siempre comparable a k B ).

La mecánica cuántica permite estas predicciones correctas de la baja capacidad calorífica porque el estado de los estados ligados es básicamente único o tiene una baja degeneración cuando se requiere que la energía esté cerca de la energía del estado fundamental. Las pequeñas perturbaciones del estado fundamental no se refieren a estados mutuamente excluyentes. En cambio, la mecánica cuántica dice que un estado que es mutuamente excluyente con el estado fundamental, para que pueda contribuir a la entropía, tiene que ser ortogonal al estado fundamental, es decir, muy diferente. Esto es equivalente a la "cuantificación del espacio de fase" habitual que se divide efectivamente en "celdas". Así es como la mecánica cuántica logra producir un "pequeño número de celdas" que se necesita, por ejemplo, para las bajas capacidades de calor.

Los defensores de la mecánica bohmiana nunca discuten ninguno de estos temas elementales, pero todavía avanzados relativamente a las "demostraciones bohmianas": equilibrio térmico de objetos, capacidad calorífica, pero también propagación de fotones y otros bosones, emisión de líneas espectrales agudas, el colapso observado de los átomos a estados propios de energía una vez que se mide la energía del fotón, la existencia de campos cuánticos fermiónicos, etc. Y la razón principal es que todas estas partes importantes de la física tal como las conocemos son irreconciliables con la mecánica de Bohm y cualquier otra teoría "realista", es decir, clásica.

Usted escribe repetidamente que los bohmianos en realidad no pueden hacer ningún cálculo. Luego escribe un montón de afirmaciones cualitativas de que la mecánica de Bohmian da las capacidades de calor incorrectas, pero usted mismo no realiza ningún cálculo. Si la teoría de Bohm da las capacidades caloríficas incorrectas, personalmente lo desafío a calcular las capacidades caloríficas en la teoría de Bohm y mostrar que difieren de lo que obtiene cuando las calcula en la mecánica cuántica.
Es fácil probar, y yo (y Dirac antes que yo) lo hemos probado, que se obtienen capacidades enormes que superan con creces O ( R ) de la mecánica cuántica (confirmada por experimentos) en cualquier teoría basada en el paradigma clásico, incluido el de Bohm. Los bohmianos probablemente no quieran ver esta prueba, y seguramente nunca la escriben ni la publican ellos mismos, pero eso no hace que la prueba desaparezca. Cuando Dirac enseñaba mecánica cuántica, todos los estudiantes sabían estas cosas básicas después de la primera lección.
La entropía de cualquier sistema clásico, es decir, realista, es el logaritmo del volumen del espacio de fase de estados que macroscópicamente se comportan de la misma manera. La teoría de Bohm incluye el espacio de fase de la función de onda malinterpretado como un campo clásico que ya da valores enormes e infinitos, y además de eso, algunas funciones adicionales, como las posiciones de las partículas. Está claro que el equilibrio producirá una entropía que es infinita e incluso cuando se agrega cualquier regularización/truncamiento, paramétricamente más alta que O ( R ) .
Bien, muéstrame el cálculo O(R). Por otro lado, por favor no lo hagas, porque sé que no se puede hacer. Ni tú ni Dirac han probado nunca tales cosas. Por otro lado, debido a que BM reproduce Phi, todos los experimentos entre las teorías en competencia deben coincidir, incluidas las capacidades caloríficas. Dirac simplemente no entendió lo que había hecho Bohm, y tú tampoco.
Las posiciones de las partículas no son posibles en la teoría de Bohm. Hablamos de esto en otra pregunta. Los beables son los campos clásicos.
Como les recordé en el otro hilo, esta elección de campos como beables falsifica instantáneamente la teoría de Bohm porque la detección puntual de fotones puede medirse experimentalmente. Una teoría solo tiene dos opciones: la ubicación del fotón que pronto se detectará puede calcularse a partir de los grados de libertad existentes justo antes de la medición (que sería el caso de la teoría de Bohm si las posiciones fueran posibles, pero admitió no puede funcionar), o el resultado se produce al azar y luego nos ocupamos de la mecánica cuántica adecuada (Copenhague). No hay una tercera opción además de Sí y No.
Muéstrame el cálculo superior a O(R) en la Mecánica Bohmiana, y nunca volveré a cuestionar la mecánica cuántica.
No se puede realizar un cálculo preciso en una teoría inexistente. Su solicitud es análoga a la solicitud de calcular la relación de masa de protones a electrones de Génesis. No existe una formulación de la mecánica de Bohm en la que se conozca el espacio de configuración completo y que admita el equilibrio térmico. Sólo se pueden realizar los cálculos con una precisión como máximo igual a la precisión con la que se formula la teoría.
Si es imposible calcularlo, entonces ¿cómo sabes que es >> O(R)?
Motl: No tengo una suscripción, por lo que no puedo ver el documento, pero aquí hay un documento con algunas de las cosas que dices que no se pueden hacer en BM. sciencedirect.com/science/article/pii/037596019190330B
La mecánica de Bohm no es " simplemente otra teoría clásica ". Es una teoría cuántica y predice conocidas desviaciones de la mecánica clásica. Y el resto de afirmaciones también son puras tonterías.
Lo siento, simplemente no es una teoría mecánica cuántica. Las teorías de la mecánica cuántica son, por definición, aquellas que obedecen a los postulados generales de QM -ver, por ejemplo , ekonomika.idnes.cz/…- y la mecánica de Bohm simplemente los viola casi todos. En cambio, obedece a los postulados generales de la física clásica. Es un ejemplo de una teoría clásica, solo una teoría clásica muy incómoda y poco natural. Es un intento fallido de evitar la QM, al igual que algunas nuevas teorías de Diseño Inteligente son esfuerzos para evitar la evolución.
@LubošMotl, No. Primero, no hay nada como " los postulados generales de QM ", los postulados de cualquier teoría física dependen de la formulación elegida, lo que se postula en alguna formulación puede ser un resultado derivado en otra formulación. En segundo lugar, la mecánica de Bohm es una formulación bien conocida de QM scitation.aip.org/content/aapt/journal/ajp/70/3/10.1119/…
Una "formulación" significa una forma equivalente de elegir las palabras, símbolos, fórmulas y grupos de leyes para describir la misma cosa . La mecánica bohmiana claramente no es equivalente a la mecánica cuántica, por lo que no puede llamarse una "formulación de la mecánica cuántica". La mecánica de Bohm es una teoría clásica vagamente definida que algunas personas proponen, de manera totalmente absurda, como reemplazo de la mecánica cuántica.
@LubošMotl, una formulación equivalente debe describir las mismas cosas, pero no debe tener los mismos postulados. No hay nada como sus "postulados generales de QM", porque cada formulación tiene su propio conjunto de postulados y resultados derivados. Como se dijo varias veces, y no voy a desperdiciar más bits en esto, la mecánica de Bohmian es una formulación bien conocida de QM. Le he dado un documento que revisa diferentes formulaciones de QM, incluido Bohm.
Puede haber conversaciones diferentes y erróneas sobre la mecánica cuántica, pero no puede haber "postulados universales diferentes" de la mecánica cuántica. El orden y la agrupación de los postulados puede depender del libro de texto, pero la carne no. Consulte cualquier introducción básica a QM y sus postulados, por ejemplo , en.wikipedia.org/wiki/… antes de volver a discutir QM. Decir que una teoría, especialmente una teoría crucial como QM, no tiene postulados ni reglas es una tontería más allá de la imaginación.
@LubošMotl, nadie habla de " ordenar y cumplir " los postulados de una formulación dada, sino de señalar que diferentes formulaciones de una teoría física pueden tener diferentes postulados. Es interesante que insista en ignorar el artículo sobre formulaciones de la mecánica cuántica, pero cite Wikipedia; es especialmente notable cuando sus propios enlaces refuta su negación de la formulación Bohmiana de QM: la formulación de onda piloto de Broglie-Bohm-Bell de la mecánica cuántica
La mecánica cuántica es una teoría. Toda teoría debe tener algunos postulados. La mecánica cuántica tiene postulados. Si rechazas la validez de los postulados, estás rechazando la teoría. ¿Qué palabra de las oraciones anteriores sigues sin entender?
@LubošMotl, nadie rechaza la validez de los postulados de una teoría física, pero señala que diferentes formulaciones de una teoría física pueden tener diferentes postulados . Avíseme si tengo que explicar lo mismo por cuarta vez.
@Luboš Motl Realmente aprecio su respuesta y comentarios. Muy útil con estructuras lógicas. Me gustaría saber su opinión sobre la función de onda de un campo de una sola partícula (sin masa o masivo) como los campos de Weyl, Dirac y similares. ¿Estas olas no son reales en absoluto? Simplemente siento que estas ondas son similares a las ondas electromagnéticas (de alguna manera) descritas por el caso sin fuente de las ecuaciones de Maxwell (si se agregan también las soluciones complejas a estas ecuaciones). Difícil creer que, por ejemplo, el campo de Dirac no es un campo real (me refiero a algo similar a un campo de fuerza...).
Estimado @J.Pak, gracias por sus amables palabras. Los campos fermiónicos (ej. Dirac, Weyl...) son completamente análogos al electromagnético pero también tienen diferencias importantes. En particular, el campo electromagnético, y todos los demás campos bosónicos, pueden ser muy fuertes y tener valores muy grandes y, por lo tanto, "clásicos". Corresponde a tener muchos fotones en el mismo estado, etc. Por otro lado, los campos fermiónicos no pueden tener vevs grandes (¡o distintos de cero!) porque como máximo 1 fermión puede estar en un estado dado debido al principio de exclusión de Pauli.
Por esta razón, los campos bosónicos pueden tener un límite clásico de campo fuerte, lo que conduce a la teoría clásica de campos. El número de fotones en cada estado posible es un número entero mucho mayor que uno y efectivamente continuo. Por otro lado, el único límite clásico posible que puede obtener de los campos fermiónicos son las partículas fermiónicas clásicas: el electrón se comporta como la mecánica clásica en algunos límites, o cosas como los líquidos de Fermi: la información se transmite por la forma del límite entre vacío y estados ocupados.

Dado que la teoría de De-Broglie Bohm esencialmente introduce una fuerza cuántica para remediar la mecánica clásica,

Eso no es lo que hace la teoría dBB. Introduce una fuerza dependiente del estado. Lo que significa que introduce estados (ondas) y luego la onda ejerce una fuerza cuántica.

¿Está el argumento de Dirac en contradicción con la teoría de Bohm?

No. Y no por las razones anteriores. La teoría dBB es para la mecánica cuántica no relativista. Y eso significa que en realidad no aborda el electromagnetismo de la misma manera que la ecuación de Schrödinger no aborda el electromagnetismo.

Cuando resuelves, digamos los niveles de energía del hidrógeno, no usas campos electromagnéticos, solo usas un potencial escalar. Incluso cuando lo resuelve como un problema de dos partículas (por lo tanto, una onda en un espacio de configuración 6d), aún escribe un potencial escalar como

V ( X mi , y mi , z mi , X pags , y pags , z pags ) = k 2 mi 2 ( X mi X pags ) 2 + ( y mi y pags ) 2 + ( z mi z pags ) 2
que es solo el potencial electrostático. Sin magnetismo, por ejemplo, y sin electrodinámica. Si tiene un potencial clásico externo, puede incluirlo, pero este no es el electromagnetismo clásico completo, en particular, no está asignando un estado al campo electromagnético, sino un estado a las partículas.

Entonces, dBB está realmente incompleto, no tiene una versión QFT completa y adecuada.

Pero todo lo que Dirac decía es que una fuerza no cambia la naturaleza que un desequilibrio en la física clásica imprime en el mundo más amplio. Pero en dBB esencialmente tienes una desviación radical de la física clásica en la que, además de una configuración, tienes un estado, una onda. Y la onda dependiente del estado actúa como una onda piloto que ejerce fuerzas dependientes del estado. Lo que te permite tener diferentes estados ejerciendo diferentes fuerzas.

Ese es el cambio de la física clásica y dado que dBB lo hace, dBB no es una teoría clásica y Dirac tiene razón, tienes que hacer algo diferente para obtener la mecánica cuántica.

Si desea explorar completamente cómo se aplican los comentarios de Dirac a dBB, debe buscar una desviación del equilibrio, es decir, un estado que no es un estado propio de energía y ver cómo evoluciona y buscar una huella. Y la ingenua ecuación de Schrödinger no relativista podría hacer que cada estado propio de energía evolucione solo una fase y, por lo tanto, por linealidad, la superposición evolucione como la suma de cada uno.

Pero cuando lanzas un potencial electromagnético externo obtienes, por ejemplo, transiciones hacia arriba desde algunos campos y transiciones hacia abajo desde otros campos. No solo evoluciona para estar permanentemente en esa superposición. Y eso se debe a que QM se convierte en QFT cuando incluye los campos de vacío y otros campos de una manera en la que esos mismos campos pueden cambiar.

Pero la teoría dBB simplemente no tiene esa parte de la mecánica cuántica porque no tiene una versión QFT. Entonces, realmente no se aplica a la situación exacta que Dirac estaba discutiendo.